esse problema e semlhante ao anterior.

2014-07-05 0:25 GMT-03:00 Ralph Teixeira <ralp...@gmail.com>:

> Estou pensando em algo com o seguinte espirito (mas tem que examinar
> todos os detalhes e ver se funciona mesmo)!
>
> 1. Suponha que f'(a) NAO EH L. Entao existe alguma sequencia (que,
> passando uma subsequencia se necessario, pode ser tomada monotona --
> vou supor spdg decrescente) z_n -> a (com z_n <>a) tal que lim
> (f(z_n)-f(a)) / (z_n-a) nao eh L.
> 2. Se a sequencia dos numeros (f(z_n)-f(a))/(z_n-a) for ilimitada,
> passe outra sub para que ele o limite dela seja +Inf ou -Inf; se for
> limitada, ela tem que ter um ponto de acumulacao que nao eh L, entao
> passe uma subsequencia para que o limite seja um numero A diferente de
> L.
>
> Em suma, neste momento temos uma sequencia z_n->a decrescente tal que
> lim (f(z_n)-f(a)) / (z_n-a) = A <> L.
>
> 3. Ideia: tome y_n=z_n. Agora, para CADA y_n fixo, vamos escolher x_n
> MUITO PERTO de a, tal que f(x_n) esteja MUITO MUITO PERTO de f(a).
> Assim, teremos algo do tipo
> [f(y_n) - f(x_n)]/[y_n-x_n] ~~~ [f(y_n) - f(a)]/[y_n-a] ~~~ A, que
> estaria **longe** de L. Pronto, isto seria uma contradicao frente aa
> hipotese dada!
>
> Vejamos os detalhes, pelo menos no caso em que A eh finito: vou
> denotar B_n=[f(y_n) - f(a)]/[y_n-a] e D=|A-L|>0.
> i) Primeiro passe outra subsequencia de forma a garantir que |B_n  -
> A| < D/4. Isto eh para garantir que este negocio estah realmente longe
> de L (e eh possivel porque o limite de B_n eh A quando n->Inf, entao
> eh soh cortar o comeco da sequencia e deixar um rabo conveniente).
> ii) Agora, para um y_n fixo, note que lim (x->a) (f(y_n) - f(x))/(y_n
> - x) = B_n. Entao, para x suficientemente proximo de a, temos
> |(f(y_n)-f(x)) / (y_n-x) - B_n |<D/4.
> iii) Entao escolha um x_n de cada vez, indutivamente, sempre no
> intervalo (x_(n-1),a) e satisfazendo (ii) (que simplesmente
> determinava um intervalo em volta de a onde x_n tinha que estar).
> iv) Pronto! A sequencia x_n eh crescente, e temos
>
> |[f(y_n) - f(x_n)]/[y_n-x_n] - L| > |A-L| - |B_n-A| - |[f(y_n) -
> f(x_n)]/[y_n-x_n] - B_n| > D - D/4 - D/4 > D/2
> e portanto o limite desta fracao ali na esquerda nao serah L, absurdo.
>
>
> (Agora falta fazer um raciocinio analogo no caso em que A=+-Inf! Mas
> tenho certeza que sai, deve ateh ser mais facil do que esse que eu
> fiz.)
>
> Abraco, Ralph.
>
> 2014-07-04 21:35 GMT-03:00 Merryl <sc...@hotmail.com>:
> > Boa noite amigos
> >
> > Obrigada a todos pela ajuda naquele outro problema.
> >
> > Gostaria de ajuda com este aqui. Já pensei mas não consegui provar.
> >
> > Seja f:I --> R contínua no ponto a do intervalo aberto I. Suponhamos que
> > para todas sequências (x_n) e (y_n) em I tais que
> >
> > (x_n) seja crescente e convirja para a
> >
> > (y_n) seja decrescente e convirja para a
> >
> > x_n < a < y_n para todo n
> >
> > exista um mesmo real L para o qual convirjam os quocientes ((f(y_n) -
> > f(x_n))/(y_n - x_n)).
> >
> > Mostre que f é diferenciável em a e que f'(a) = L
> >
> > Eu tentei partir o quociente acima em duas partes, escrevendo-o como
> >
> > (f(y_n) - f(a))/(y_n - a) (y_n - a)/(y_n - x_n) - (f(x_n) - f(a))/(x_n -
> a)
> > (x_n - a)/(y_n - x_n)
> >
> > Mas como a hipótese é que f é só contínua em a, não se assume
> > diferenciabilidade, isto não permite chegar a f'(a) = L.
> >
> > Obrigada
> >
> > --
> > Esta mensagem foi verificada pelo sistema de antivírus e
> > acredita-se estar livre de perigo.
>
> --
> Esta mensagem foi verificada pelo sistema de antivírus e
>  acredita-se estar livre de perigo.
>
>
> =========================================================================
> Instru�ões para entrar na lista, sair da lista e usar a lista em
> http://www.mat.puc-rio.br/~obmlistas/obm-l.html
> =========================================================================
>

-- 
Esta mensagem foi verificada pelo sistema de antiv�rus e
 acredita-se estar livre de perigo.

Responder a